Which equations can be used to find the lengths of the legs of the triangle? Select three options. 0.5(x)(x + 2) = 24 x(x + 2) = 24 x2 + 2x – 24 = 0 x2 + 2x – 48 = 0 x2 + (x + 2)2 = 100

Answers

Answer 1

By solving a linear equation, it can be calculated that-

First fourth and fifth option are correct

What is a linear equation?

At first it is important to know about linear equation

Equation shows the equality between two algebraic expressions by connecting the two algebraic expressions by an equal to sign.

A one degree equation is known as linear equation.

Length of two sides of a right angled triangle = x, x+2

Area = [tex]\frac{1}{2}\times x \times (x + 2)[/tex]

By the problem,

The linear equation is

[tex]\frac{1}{2}\times x \times (x + 2) = 24\\[/tex]

0.5x(x + 2) = 24

First option is correct

x(x + 2) = 48

[tex]x^2 + 2x - 48 = 0[/tex]

Fourth option is correct

For the sixth option,

[tex]x^2 + (x + 2)^2 = 100\\x^2 + x^2 + 4x + 4 - 100 = 0\\2x^2 + 4x - 96 = 0\\2(x^2 + 2x - 48) = 0\\x^2 + 2x - 48 = 0[/tex]

Which is the third option.

First fourth and fifth option are correct

To learn more about linear equation, refer to the link-

https://brainly.com/question/2030026

#SPJ1


Related Questions

geometry midterm coming up and i need help on similar figures.

Answers

A triangle is a polygon with three edges and three vertices. It is one of the basic shapes in geometry. A triangle with vertices A, B, and C is denoted triangle ABC.

By Pythagoras' theorem,

(20)²+(48)²=x²

400+2304=x²

2704=x²

x=52

As we know, area of a triangle = 1/2 × b×h

                                           480= 1/2× 52 ×h

                                                  h=2×480/52

                                                  h= 18.46 = BD          

What is geometry?

Geometry with arithmetic is one of the oldest branches of mathematics. It deals with spatial properties such as distance, shape, size, and relative position of shapes. Mathematicians working in the field of geometry are called geometers.

Until the 19th century, geometry was mostly devoted to Euclidean geometry, which included the concepts of point, line, plane, distance, angle, area, and curve as basic concepts.

In the 19th century, several discoveries dramatically expanded the scope of geometry. One of the oldest discoveries of this kind is Carl Friedrich Gauss's Theorem Egregium ("Remarkable Theorem"), which loosely asserts that the Gaussian curvature of a surface is independent of any particular embedding in Euclidean space. I'm here. This meant that surfaces could be studied intrinsically, i.e., as spaces themselves, and extended to manifold theory and Riemannian geometry.

Therefore, A triangle is a polygon with three edges and three vertices. It is one of the basic shapes in geometry. A triangle with vertices A, B, and C is denoted triangle ABC.

To know more about geometry, visit:

https://brainly.com/question/24375372

#SPJ1

The coordinates of C are ( 4 , 4 ) , and the midpoint of CD ¯ is at M ( - 2 , - 6 ) . What are the coordinates of point D

Answers

Answer:

D (-8, -16)

Step-by-step explanation:

(4+x)/2 = -2

4+ x = -2 · 2

4 + x = -4

x = -4 - 4

x = -8


(4 + y)/2 = -6

4 + y = -6 · 2

4 + y = -12

y = -12 - 4

y = -16

In the United States about 7% of the male population and about 0.4% of the female population is red-green color blind ( that is they cannot distinguish red from green, or see red and green differently from how others do). The population in the United States consist of about 49% males and 51% females. A person is randomly selected.
Let M be the event that the person is male, let F be the event that the person is female, let CB be the event that the person is red-green color blind, and let NC be the event that the person is not red-green color blind
A. Find P(CB \mid M)
B. Find P(CB \cap M)
C. Find P(CB)
Find P(M \mid CB)

Answers

Ans (A)   P(CB/M) is  0.07

Ans (B)   P(CB ∩ M) is 0.0343

Ans (C)   P(CB) is 0.03634

Ans (D)   P(M/CB) is 0.94386

Let the population be 100.

Therefore,

No. of males (M) = 49

No. of females (F) = 51

No. of colourblind males =

[tex]\frac{7}{100} * 49 = \frac{343}{100} = 3.43[/tex]

No. of colourblind females =

[tex]\frac{0.4}{51} * 100 = 0.204[/tex]

Total colourblind people (CB) = 3.43 + 0.204 = 3.634

Ans (A)

P(CB/M) = P(CB ∩ M)/P(M) = 3.43/49 = 0.07

Ans (B)

P (CB ∩ M) = 3.43/100 = 0.0343

Ans (C)

P(CB) = 3.634/100 = 0.03634

Ans (D)

P (M/CB) = P(M ∩ CB)/P(CB) = 3.43/3.634 = 0.94386

These are the answers.

To learn more about conditional probability,

brainly.com/question/27684587

#SPJ4

HELPP!!!!! HURRY!!!! Function g(x) = |x − 9| is a transformation of function f(x) = |x|. What effect does the value 9 have on the graph of function f? A. It translates the graph 9 units to the left. B. It translates the graph 9 units to the right. C. It vertically compresses the graph by a factor of 9. D. It vertically stretches the graph by a factor of 9.

Answers

Answer:

C

Step-by-step explanation:

Answer:

B: It translates the graph 9 units to the right.

Step-by-step explanation:

The value 9 has the effect of translating the graph of function f(x) = |x| 9 units to the right. This is because the transformation g(x) = |x - 9| is obtained from f(x) = |x| by subtracting 9 from the argument of the absolute value function. This has the effect of shifting the graph of f(x) to the right by 9 units.

Therefore, the correct answer is B: It translates the graph 9 units to the right.

Find the inverse of the equation
Y=5/x-1

Answers

The inverse of the equation is y = (5/x) + 1

How to determine the inverse of the equation?

From the question, we have the following parameters that can be used in our computation:

y = 5/(x - 1)

Start by swaping the positions of x an dy

So, we have the following representation

x = 5/(y - 1)

So, we have

x(y - 1) = 5

Divide both sides by x

y - 1 = 5/x

Add 1 to both sides

y = (5/x) + 1

Hence, the inverse is y = (5/x) + 1

Read more about inverse equation at

https://brainly.com/question/3831584

#SPJ1

A set of data items is normally distributed with a mean of 300 and a standard deviation of 60. Why’d the data item in this distribution that corresponds to the given z-score
Z= 2.5

Answers

The data item that corresponds to the z-score of 2.5 is 450.

What is Standard deviation?

The standard deviation is a statistic that expresses how much variance or dispersion there is in a group of numbers. While a high standard deviation suggests that the values are dispersed over a wider range, a low standard deviation suggests that the values tend to be close to the mean (also known as the anticipated value) of the collection.

A random variable, sample, statistical population, data set, or probability distribution's standard deviation is equal to the square root of its variance. Although less robust in practice, it is algebraically easier than the average absolute deviation. The fact that the standard deviation is expressed in the same unit as the data, as opposed to the variance, makes it a valuable statistic.

As, x = z×s + u, where z=2.5, s=60, u=300

⇒ x = 2.5×60 + 300

⇒ x = 150 + 300

⇒ x = 450

The data item that corresponds to the given z-score is 450.

To know more about standard deviation visit:

brainly.com/question/23907081

#SPJ1

Let Y1 and Y2 be independent random variables, both uniformly distributed on (0, 1). Find the probability density function for U = Y1Y2.

Answers

The probability density function as calculated from the given data is       fₙ (u)=−ln(u).

Y1 and Y2 are independent random variables and both have uniform distribution over the interval (0,1)(0,1).

We need to find out the density function of  U = Y1Y2 using the method of transformations.

Density function of Y1

= 0 or 1

Density function of Y2

= 0 or 1

Therefore joint density function is,

( 0 , 1)   where, 0≤y 1 ≤1,0≤y 2 ≤1

There are 3 steps to find joint density function of Y and U using methods of transformations

Hence, the required probability density function is :

fₙ (u)=−ln(u)

An independent random variable is one that has no effect on the other random variables in your experiment. In other words, it has no bearing on the likelihood of another occurrence occurring.

To learn more about independent random variables

https://brainly.com/question/28464931

#SPJ4

Information for Questions 5-7: The Annual Survey of Colleges in the U.S. also reports on the average amount of student loan debt by state. The most recent data by state showed an approximately normal shape with a minimum average amount of
$19,000
and a maximum average amount of
$38,000
. Use the minimum and maximum value and the normal shape to estimate the mean and standard deviation of the data for the 50 states. (Mean and standard deviation have been rounded to the nearest
$100
.) a. mean
=$25,000;
standard deviation
=$3,500
b. mean
=$26,800;
standard deviation
=$2,100
c. mean
=$26,800;
standard deviation
=$3,700
d. mean
=$28.500;
standard deviation
=$1,800
e. mean
=$28,500;
standard deviation
=$3,200
A federal study of student loan debt plans to focus on those states with the highest amounts of student loan debt. The study plans on targeting those states that fall in the top
2.5%
in terms of average student loan amounts. What is the cutoff point that would place a state in the top
2.5%
based on student loan debt? (Use the mean and standard deviation estimates from Question #5.) a.
$38,100
b.
$34,900
c.
$31,700
d.
$32,500
e.
$22,100

Answers

The correct answer is $34900.

X : Average amount of loan of student debt by state .

X = N ( μ = $28500 , σ² = ( 3200 )² )

To find : What is the cutoff point that would place a state in top 2.5%

Let it be X

P( X > x ) = 0.025

P(X-μ/σ > X - 28500/3200) = 0.025

P(z >Z ) = 0.025

1 - P(z<Z ) = 0.025

P(z<Z ) = 0.975

φ(z) = 0.975

Z = φ⁻¹(0.975)

X - 28500/3200 = 1.96

X = 34772

i.e; P(X > 34772) = 0.025

Since, it is nearly equal to $34900 option b is correct.

To learn more about cutoff point click here:

brainly.com/question/29031557

#SPJ4

Choose the number that is greater than:
4.526 x 10^6
OA) 5.79 x 105
OB) 9.99 x 105
OC) 4.123 x 106
OD) 55,264,900

Answers

Answer:   C) 4.123 x 106

Step-by-step explanation:

you are designing a rectangular wooden box with width 4 inches greater than its height, and length 3 times its height. the box has wood that is 1 inch thick on each side of the four sides on the top and bottom. a. write a polynomial function ox in standard form for the volume of the rectangular prism formed by the outer surfaces of the box.

Answers

O (x) = [tex]3x^{3} + 12x^{2}[/tex] is  a polynomial function ox in standard form for the volume of the rectangular prism formed by the outer surfaces of the box

what is polynomial function ?

Using mathematical operations like addition, subtraction, multiplication, and division, a polynomial is an equation made up of variables, constants, and exponents (No division operation by a variable)

calculation

outside length are height = x

width = x +4 , length = 3x

volume = l *b * h

o(x) = 3x ( x+ 4 )x

= 3[tex]x^{2}[/tex](x+4 )

= O (x) = [tex]3x^{3} + 12x^{2}[/tex]

there is 1 inch of wood , so lengths of inner sides 2 inch less than outer sides as 1 inch from reduced

length = 3x-2

width = x +4-2 = x+2

height = x -2

volume = l*b*h

=[tex]3x^{3} - 2x^{2} - 12x + 8[/tex]

w(x) = [tex]14(6^{2}) + 12(6) -8\\ 504 + 72 -8 \\568inch^{3}[/tex]

so volume of wood = [tex]568inch^{3}[/tex] for x =6

O (x) = [tex]3x^{3} + 12x^{2}[/tex] is  a polynomial function ox in standard form for the volume of the rectangular prism formed by the outer surfaces of the box.

To know more about polynomial function visit :-

https://brainly.com/question/12976257

#SPJ4

URGENT! Please help with the image attached!

Answers

The missing angle in the parallel lines is as follows;

m∠STW  = 125 degrees

How to find the angles when parallel line are cut by a transversal?

When parallel lines are cut by a transversal, angle relationships are formed such as corresponding angles, alternate interior angles, alternate exterior angles, linear angles, vertically opposite angles etc.

Therefore, let find angle m∠STW .

VX and SU are parallel lines. RY is the transversal line.

Hence,

m∠VWY = 125 degrees

m∠VWY =  m∠STW (corresponding angles)

Corresponding angles are congruent to each other.

Therefore.

m∠STW  = 125 degrees

learn more on angles here: https://brainly.com/question/18359073

#SPJ1

Evaluate the function when x = 3, x = 0, and x = -4. (Lesson 4.8)
14. f(x) = -4x + 3
12. h(x) = − 8x
13. g(x) = 5x - 9
16. h(x) = 1.4x
15. g(x)=-3x - 12
17. f(x) = 1/4x

12-17 please

Answers

Just plug down the numbers 3,0,-4 on all the problems to get the answer. If you need help feel free to ask

Katy has 5 white flowers, x red flowers and (2x+1) yellow flowers.
She picks a flower at random.
The probability that it is white is 1/12
Find the probability that it is yellow.

Answers

Answer:

5/12

Step-by-step explanation:

Total 12 flowers

5 are white

12-5=7

red + yellow = 7

x+2x+1 = 7

3x+1=7

3x=6

x=2

white : 5

red : 2

yellow : 5

12 flowers in total, 5 yellow flowers, probability that yellow is picked is 5/12

subtract and reduce to lowest terms

Answers

The answer is none of the answers are correct. First change the fractions to 24/7 and 11/5. Upscale 24/7 by 5 and 11/5 by 7. That gives 120/35 - 77/35. That is equal 43/35, which is 1 8/35

Answer:

Below

Step-by-step explanation:

To add or subtract fractions ( or numbers with fractions in them) you HAVE to put the fractions into a common denominator form .....

Common denominator for 5 and 7 is 35

3 3/7 = 120/35

2  1/5 = 77/35

120/35  -  77/35  =  ( 120 - 77) / 35 =  43/35 = 1  8/35

Select the table that represents a liner function. (Use graph if necessary)

Answers

The table that represents a linear equation is the first one, and the line is:

y = 5x + 5

Which table represents a linear function?

To check which table represents a linear function, you need to identify the increases in x, and check that al the increases/decreases in y are constant.

For example, if you look at the last table:

x = 0, 1, 2, 3

y = 1, 2, 4, 8

First x goes from 0 to 1, so 1 unit increase.

y goes from 1 to 2, 1 unit increase.

Then x goes from 1 to 2, 1 unit increase.

y goes from 2 to 4, 2 units increase.

The increase is not constant, thus it is not a linear equation.

The tables 2 and 3 can be discarded because have both increases/decreases.

Table 1 is the one that represents a linear equation:

x: 0, 1, 2, 3

y: 5, 10, 15, 20

Where the increases in y are always of 5 units, and the linear equation is:

y = 5*x + 5

Learn more about linear equations:

https://brainly.com/question/1884491

#SPJ1

The cost of a plumber consists of two parts: the fixed costs and the hourly rate.
One piece of work takes the plumber 5 hours and costs £155.
Another piece of work takes the plumber 8 hours and costs £230.
(a) i) What is the fixed cost of the plumber?
ii) What is the hourly rate for the plumber?

Answers

The fixed cost of the plumber is £30 and the hourly rate for the plumber is  £25.

What is an equation?

Mathematical expressions with two algebraic symbols on either side of the equal (=) sign are called equations.

This relationship is illustrated by the left and right expressions being equal. The left-hand side equals the right-hand side is a basic, straightforward equation.

As per the given information in the question,

Let the fixed cost of a plumber is y.

Let the number of hours of work done be x.

As per the first condition, that one piece of work takes the plumber 5 hours and costs £155.

Then, the equation according to the statement will be,

y + 5x = £155      (i)

In the second condition, another piece of work takes the plumber 8 hours and costs £230.

Then, the equation according to the statement will be,

y + 8x = £230     (ii)

Now, subtract equation (ii) from the equation for the value of x,

y + 5x = £155

y + 8x = £230

-3x = -75

x = £25

Put x = £25 in equation (i)

y + 5x = £155

y + 5(25) = 155

y = 155 - 125

y = £30.

To know more about equation:

https://brainly.com/question/29657983

#SPJ1

I’m confused on this question

Answers

Answer: 3 of the squares should go down then 8 to the right.

Step-by-step explanation:


it just wants you to say how it was translated so take the corresponding points like p and p’ then count of many tiles it move to the right and then down to get from p to p’ you’ll see that it’s 8 right and 3 down.


hope this helps!

Which functions have a vertex with a x-value of 0? Select three options.
Of(x) = |x|
f(x) = x + 3
f(x) = 1x + 31
f(x) = 1x1-6
Of(x) = x + 31-6

Answers

The functions that have a vertex at x = 0 are (a) f(x) = |x| and (d) f(x) = |x| - 6

How to determine the function from the vertex

From the question, we have the following parameters that can be used in our computation:

The absolute functions in the options

As a general rule;

A absolute functions can be represented as

f(x) = a|x - h| + k

Where

Vertex = (h, k)

A vertex that has x value of 0 means

(h, k) = (0, k)

So, we have

f(x) = a|x - 0| + k

Evaluate

f(x) = a|x| + k

Looking at the options, we have

f(x) = |x| and f(x) = |x| - 6

Read more about absolute functions at

https://brainly.com/question/10538556

#SPJ1

A recipe uses 1 1/4 cups of milk to make 10 servings. If the same amount of milk is used for each serving, how many servings can be made using 2 gallons of milk?

Answers

Answer:

what we cookin ma boi

Step-by-step explanation:

A manager at a local bank analyzed the relationship between monthly salary (y, in $) and length of service (x, measured in months) for 30 employees. She estimates the model:
Salary = β0 +β1 Service + ε. The following ANOVA table below shows a portion of the regression results.
df SS MS F
Regression 1 555,420 555,420 7.64
Residual 27 1,962,873 72,699 Total 28 2,518,293 Coefficients Standard Error t-stat p-value
Intercept 784.92 322.25 2.44 0.02
Service 9.19 3.20 2.87 0.01
Which of the following is the monthly salary of an employee that has worked for 48 months at the bank?
rev: 11_17_2017_QC_CS-109762
$441.
$785.
$1,050.
$1,226.

Answers

The monthly salary of an employee that has worked for 48 months at the bank is $1226.04. So the option d is correct.

The given table is:

                          Df          SS                     MS                F

Regression          1          555,420          555,420          7.64

Residual              27       1,962,873         72,699

Total                    28        2,518,293

                    Coefficients         Standard Error         t-stat        p-value

Intercept          784.92                322.25                    2.44           0.02

Service             9.19                     3.20                      2.87           0.01

We have to find the monthly salary of an employee that has worked for 48 months at the bank.

Salary = β(0)+ β(1) Service

y = β(0)+ β(1)x

where, Service = x(in months)

Service = y(in $)

From the table

y = 784.92+9.19x

If x=48 months

Then, y = 784.92+9.19(48)

y = 784.92+441.12

y = 1226.04

Hence, the monthly salary of an employee that has worked for 48 months at the bank is $1226.04. So the option d is correct.

To learn more about t statistic link is here

brainly.com/question/15236063

#SPJ4

HW9.4. Algebraic and geometric multiplicity of eigenvalues Observe that -1 is an eigenvalue of A= [-1.00 0.00 1.00 1.00 0.00 0.00 0.00 1.00 -2.00 Determine the algebraic and geometric multiplicity of this eigenvalue of A. Algebraic multiplicity integer Geometric multiplicity integer

Answers

The value of algebraic multiplicity will be 1 and geometric multiplicity will be 1

[tex]$$\begin{aligned}& A=\left[\begin{array}{ccc}3 & -2 & 0 \\-2 & 4 & 4 \\1 & 0 & 2\end{array}\right] \\& \Rightarrow A=\left[\begin{array}{ccc}-1 & 1 & 0 \\0 & 0 & 0 \\1 & 1 & -2\end{array}\right]\end{aligned}$$[/tex]

[tex]$$Det(A-\lambda I) = Det A=\left[\begin{array}{ccc}(-1-\lambda) & 1 & 0 \\0 & -\lambda & 0 \\1 & 1 & (-2-\lambda)\end{array}\right]$$[/tex]

[tex]$$-(1+\lambda)\left|\begin{array}{cc}-\lambda & 0 \\1 & -2-\lambda\end{array}\right|-1\left|\begin{array}{cc}0 & 0 \\1 & -2-\lambda\end{array}\right|$$[/tex]

[tex]$$\begin{aligned}& =-(1+\lambda)\left(\lambda^2+2 \lambda\right) . \\& =-\lambda(1+\lambda)(\lambda+2) .\end{aligned}$$[/tex]

So, for [tex]\lambda=-1[/tex], eigen values are [tex]\lambda[/tex]=0,-1,-2.

Hence, the value of algebraic multiplicity=1

[tex]AV=\lambda v \\& \Rightarrow A V=-V\\ \Rightarrow-V_1+V_2=-V_1\\ \Rightarrow V_2=0 \\& v_1+v_2-2 v_3=-v_3\\ \Rightarrow v_1+v_2-v_3=0\\ \Rightarrow v_1=v_3 \\[/tex]

[tex]$$\begin{aligned}& \Rightarrow \quad\left[\begin{array}{l}v_1 \\0 \\v_1\end{array}\right] \\& =v_1\left[\begin{array}{l}1 \\0 \\1\end{array}\right]\end{aligned}$$[/tex]

So, the value of eigen vectors are: [tex]& =\left[\begin{array}{l}1 \\ 0 \\ 1\end{array}\right] \\[/tex]

And the value of geometric multiplicity =1

For more questions on Geometric and algebraic multiplicity:

https://brainly.com/question/29584270

#SPJ4

. Cell Phone Models A particular cell phone company
offers 4 models of phones, each in 6 different colors and
each available with any one of 5 calling plans. Howy
many combinations are possible?
:C

Answers

120 many combinations are possible.

What is combination formula?

The number of possible ways to choose items from a collection when the order of choice is irrelevant is determined using the combination formula. Combination, to put it simply, is the process of choosing items or things from a bigger group where order is irrelevant.

Consider a collection of three integers P, Q, and R. The number of ways that we may choose two numbers from each group is thus determined by combination.

It is represented as: [tex]^nC_r[/tex]

where, n = total number of objects in the set

r = number of choosing objects from the set

Given that,

Cell Phone Models A particular cell phone company offers:

4 models of phones

each in 6 different colors

each available with any one of 5 calling plans

Thus, possible combination numbers are:

= (No. of models) × (No. of colors) × (No. of calling plans)

= (4)× (6)× (5)

= 120 possibilities

To know more about combinations refer to:

https://brainly.com/question/28065038

#SPJ1

Need answers for all parts.

Answers

(a) The balance be at the end of the year is $502.53.

(b) The Effective annual interest rate is 0.5%.

(c) The Effective annual interest rate must be less than 6%.

What is effective annual interest rate?

The percentage of interest on a loan or financial product that would apply if compound interest built up over a year without any payments would be called the effective interest rate, also known as the effective annual interest rate, annual equivalent rate, or simply effective rate.

Given:

P = $500,  n = 12,  r = 0.5% = 0.005, t = 1

So, the balance be at the end of year is,

[tex]A = P(1+\frac{r}{n})^n^t \\A = 500(1+\frac{0.005}{12})^1^2^*^1 \\A = 500(1.00042)^1^2\\A = 502.53[/tex]

Now to find the effective annual interest rate,

Effective annual interest rate = [tex]= (1+\frac{r}{n})^n-1 = (1+\frac{0.005}{12})^1^2 - 1= 0.005[/tex]

Effective annual interest rate = 0.5%

So, the Effective annual interest rate must be less than 6%.

Hence,

(a) The balance be at the end of the year is $502.53.

(b) The Effective annual interest rate is 0.5%.

(c) The Effective annual interest rate must be less than 6%.

To know more about the Effective annual interest rate, click on the link

https://brainly.com/question/18234486

#SPJ1

Please help will mark Brainly

Answers

Answer:

Below

Step-by-step explanation:

The first set of graphs is the correct one....it shows Y = constant 18

and shows Z as starting  at 10 (deposit) THEN INCREASING 2 per hour....

  where they cross is 4 hours and charges are the same

8 ft 10 ft Find the area of this figure. Round your answer to the nearest hundredth. Use 3.14 to approximate . A = [?]ft? Notice that only half of the circle is included in the figure!​

Answers

find the area of the rectangle;

w x l = 8 x 10 = 80ft^2

circle;

pi(r)^2

r = 4

pi(4)^2

pi(16) = 50.24

80 + 50.24 = 130.24

Your family ate a delicious meal at Jimmies of Savin Rock. The meal itself cost $147.25, you want to leave 20% tip and there was 7.35% tax added

Answers

Answer:

27.29 honestly this is a wild guess

Step-by-step explanation:

7.35% of 147.25 = 10.82

147.25-10.82=136.43

20% of 136.43 = 27.286(27.29)

Prior to the current period, Benjamin Rubinek, whose tax return filing status is single, had earnings subject to Medicare tax of $199,500. During the current week, Benjamin has gross earnings of $2,900, and he requests that 7% of gross earnings be contributed to a 403(b) plan. Benjamin's employer will withhold $ in Additional Medicare Tax for this period.

Answers

Benjamin's employer will withhold $42.05 in Additional Medicare Tax for this period.

How much will be withheld by the Medicare Tax?

To calculate the amount withheld by Benjamin's employer in Medicare tax, we apply the proportion of the Medicare tax to his total earnings.

The parameters for this problem are given as follows:

Medicare Rate: 2.9%, however, only 1.45% of this is withheld by the employee.Benjamin's weekly earnings: $2,900.

Then the amount withheld by the employee for the Medicare Tax is calculated with the multiplication of the decimal equivalent of 1.45%, which is of 0.0145, by Benjamin's gross earnings of $2,900 as follows:

Amount = 0.0145 x 2900 = $42.05.

More can be learned about proportions at https://brainly.com/question/24372153

#SPJ1

To find the slope of a curve at a given point, we simply differentiate the equation of the curve and find the first derivative of the curve, i.e., dy/dx.

Answers

To find the slope of the curve, first differentiate the equation of the curve and substitute the value of x in the result

The slope of the curve is the change in y coordinates with respect to the change in x coordinates of the line

To find the slope of the curve at a given point

First differentiate the given equation of the curve with respect to x

That is dy / dx.

The derivative of the equation of the curve  is the slope of the curve.

In next step substitute the value of x in the slope of the curve

The result will be the slope of the curve at a given point

Therefore, these are the steps to find the slope of the curve

I have answered the question in general, as the given question is incomplete

The complete question is:

How to find the slope of a curve at a given point?

Learn more about slope of the curve here

brainly.com/question/4748565

#SPJ4

Parallel and Perpendicular lines Block

Answers

By observing the figure we can make the pairs as

∠1 ≅ ∠3 ---------(Opposite angles)
∠2 ≅ ∠4 ---------(Opposite angles)
∠5 ≅ ∠7 ---------(Opposite angles)
∠6 ≅ ∠8 ---------(Opposite angles)
∠4 ≅ ∠6 ---------(Alternate angles)
∠3 ≅ ∠5 ---------(Alternate angles)

What are opposite angles and Alternate angles?

Opposite angles are angles that are located opposite each other, with respect to a straight line or a transversal. They are also called vertically opposite angles.

Alternate angles are angles that are located on opposite sides of the transversal and on the same side of the straight line or the transversal. They are also called corresponding angles.

In the given figure,

By observing the opposite angles and alternate angles we can fill the blocks,

∠1 ≅ ∠3 ---------(Opposite angles)

∠2 ≅ ∠4 ---------(Opposite angles)
∠5 ≅ ∠7 ---------(Opposite angles)
∠6 ≅ ∠8 ---------(Opposite angles)


∠4 ≅ ∠6 ---------(Alternate angles)

∠3 ≅ ∠5 ---------(Alternate angles)

Hence, we have found the opposite angles and alternate angles from the figure.

To learn more about opposite angles and alternate angles, visit:

https://brainly.com/question/26167358

#SPJ1

Consider the drawing and information below. The drawing is NOT to scale.
1.What is m < 2 show work and calculations.
2. What is m < 4 show your work and calculations.​

Answers

Answer:

m∠2 = 110°

m∠4 = 110°

Step-by-step explanation:

Corresponding Angles Postulate

When a straight line intersects two parallel straight lines, the resulting corresponding angles are congruent.

Vertical Angles Theorem

When two straight lines intersect, the opposite vertical angles are congruent.

Transitive Property of Equality

If any angles are congruent to the same angle, then they are congruent to each other.

--------------------------------------------------------------------------------------

As line p is parallel to line q, according to the Corresponding Angles Postulate:

⇒ m∠2 = m∠10

According to the Vertical Angles Theorem:

⇒ m∠10 = m∠13

According to the Transitive Property of Equality:

⇒ m∠2 = m∠13

Therefore:

⇒ 8x + 14 = 10x - 10

⇒ 8x + 14 - 8x = 10x - 10 - 8x

⇒ 14 = 2x - 10

⇒ 14 + 10 = 2x - 10 + 10

⇒ 24 = 2x

⇒ 2x = 24

⇒ 2x ÷ 2 = 24 ÷ 2

⇒ x = 12

Substitute the found value of x into the expression for the measure of angle 2:

⇒ m∠2 = 8(12) + 14

⇒ m∠2 = 96 + 14

⇒ m∠2 = 110°

--------------------------------------------------------------------------------------

As line p is parallel to line q, according to the Corresponding Angles Postulate:

⇒ m∠12 = m∠4

⇒ 15y + 5 = 13y + 19

⇒ 15y + 5 - 13y = 13y + 19 - 13y

⇒ 2y + 5 = 19

⇒ 2y + 5 - 5 = 19 - 5

⇒ 2y = 14

⇒ 2y ÷ 2 = 14 ÷ 2

⇒ y = 7

Substitute the found value of y into the expression for the measure of angle 4:

⇒ m∠4 = 13(7) + 19

⇒ m∠4 = 91 + 19

⇒ m∠4 = 110°

Other Questions
an activity ratio that measures how many times the inventory is turned over to meet the total sales figure is called the inventory turnover. a. true b. false WILL GIVE BRAINLIEST!The aquarium has one fewer redfish than blue fish. 60% of the fish are blue. How many blue fish are in the aquarium? Show your work. PLEASE HELP ME ITS DUE TONIGHT PLEASEEEE1.How do you think Murrays experiences during segregation may have impacted and influenced her?2.Why was Murrays family worried about her applying to UNC? What does this tell you about the society inwhich African Americans were living at the time? determine whether or not the distribution is a discrete probability distribution and select the reason why or why not. X = 0, 1, 9P( X = x ) | 0.41 | 0.6 | 0.06 100 POINTSFind the focus. x^2 = 6y (please explain, I want to understand the solution) when a political party makes a(n) for an elected office, it selects a person to run as the party's candidate in the general election for that office. How do quotas affect pricesA)They keep them highB)They lower themC)They keep them the sameD)They do not change price consider the interaction between labor and capital within the production process and answer the following questions. how do economists refer to the amount of capital employed per worker? nitrogen is a nutrient needed by plants for growth. nitrogen is naturally cycled into the soil where it is rapidly absorbed by plants. what would a farmer most likely do to replenish nitrogen that was depleted from the soil? true or false: we can determine if a star is moving toward or away from earth by measuring its brightness. Write a quadratic function in standard form whose graph passes through (-5,0), (0,-8), and (4,0) Use the drop-down menu to choose a synonym for the word mediate as it would be used in this sentence. In the context of the ways in which consumers are treated, competition in the marketplace serves to protect consumers. a. True b. False. when hiring personnel, the profamuch pharmaceutical company communicates their security policy in regards to ongoing organizational efforts of new drug compounds. what security document is typically required to be signed by its new employees that would address the sensitivity of the new drug compounds Fabio and Carlos play on a basketball team together. In the last game, Fabio had 7 points less than 2 times as many points as Carlos. Fabio scored 31 points in the game.How many points did Carlos score? assuming the size of the packets is equal, why the efficiency of pure aloha is worse than slotted aloha? a. because packets can collide partially without time boundaries in pure aloha, resulting in more collisions. b. because nodes in slotted aloha can detect collisions. c. because pure aloha has more empty slots without time synchronization. Given f(x) = x + 4x, find the average rate of change of f(x) on the interval [4, 4+h]. Your answer willbe an expression involving h. find qotation sthat support the following points about the old mans relashonship with marlin he has caught. he in intrigued by ti g if a certain radioactive substance has a half-life of 7 days, how long, in days, does it take for the sample to decay to 1 3 of its original amount jackson opened a furniture store as a sole proprietor. during the recession, business declined and he was not able to pay his suppliers and creditors. due to unilimited liability